PDA

View Full Version : Faster than light travel within Sci-Fi



Maryring
2013-06-18, 03:58 PM
What would be the hardest way (as in "hard science vs soft science". Most realistic and closest to possible by real world physics) to travel faster than light? Am trying to write some science fiction, and I want to keep it as rooted in actual real world sciences as possible. But I still need a big starship traveling from one star system to another.

Mando Knight
2013-06-18, 04:20 PM
The Alcubierre drive (http://en.wikipedia.org/wiki/Alcubierre_drive) is probably the closest thing to a plausible FTL system. It requires negative energy and exotic matter to exist, however, and we don't quite have concrete evidence of either (as far as I know).

snoopy13a
2013-06-18, 04:42 PM
There is always the option of not addressing it.

For example, when you read a contemporary novel, the author does not explain the physics of flight when a character rides in a plane. Nor does an author explain the intricacies of an internal combustion engine when a character drives a car.

This makes even more sense if your point-of-view characters are ignorant of the technology themselves. Take a cell phone, for instance. Most people use one, but can most people explain how they work? How many street thugs know the physics behind a handgun? Very few, but many know to operate one.

Even if the characters are knowledgeable, it doesn't mean that they have to discuss the technology. Two aerospace engineers carpooling to work at Boeing may be talking about the Mariners or intra-office politics instead of how to improve some component of a jet engine. If a mechanic is told to fix a light speed engine, the author need not go into details, but rather just that the engine is being fixed.

Palanan
2013-06-18, 05:04 PM
Originally Posted by snoopy13a
There is always the option of not addressing it.

There are plenty of authors who take this approach, to one degree or another. Sometimes it devolves into Star Trek technobabble, in other cases merely glossed over with the lightest of references. Orson Scott Card tends to do the latter a fair amount; he focuses on characters and their stories much more than the technology they use.

C.J. Cherryh once said that FTL was her one "freebie," meaning she was rigorous about most other elements of her Alliance/Union universe, but allowed herself FTL without a detailed explanation simply because the stories she wanted to tell, both human and alien, required it to exist.

Also, if you haven't already, I recommend you pick up The Starflight Handbook (http://www.amazon.com/Starflight-Handbook-Pioneers-Interstellar-Editions/dp/0471619124/), which is a classic by now and well worth a careful read--it's absolutely wonderful and inspiring. They deal primarily with sublight propulsion, since it's by far more feasible (as these things go) but I think they give a nod to FTL at the end.

(I'd check to be sure, but my copy has apparently gone missing, which makes me suddenly, deeply glum.)

Soras Teva Gee
2013-06-18, 05:19 PM
As mentioned the Alcubierre drive is the most coherent proposal for maybe just maybe actually getting apparent FTL in reality.

There's also the old standbye of wormholes/fold drives if you don't fuss on the details and energy requirements too much. Since its exploiting a principle that nominally exists in science.

After that hyperspace demands relatively few additions to current scient depending on how exactly you do it. For a fairly rigorous and detailed (if not that scientific) example check out the Honor Harrington series.

noparlpf
2013-06-18, 09:57 PM
What would be the hardest way (as in "hard science vs soft science". Most realistic and closest to possible by real world physics) to travel faster than light? Am trying to write some science fiction, and I want to keep it as rooted in actual real world sciences as possible. But I still need a big starship traveling from one star system to another.

The hardest way? Running. :smalltongue:

Seriously? Others have mentioned the Watchacallit Drive. Personally, despite my love of minute details, I'd rather just gloss over it.

warty goblin
2013-06-18, 11:30 PM
The hardest way? Running. :smalltongue:

Seriously? Others have mentioned the Watchacallit Drive. Personally, despite my love of minute details, I'd rather just gloss over it.

Ayup. Comparing the hardness of FTL methods in sci-fi is rather like judging which type of gelatin best survives having cinder blocks dropped on it from great height: ultimately the differences are irrelevant to the big, sticky mess the enterprise makes out of everything. Just go with whatever flavor you find works best with your other ingredients.

Mando Knight
2013-06-18, 11:59 PM
Strictly speaking, of course, the most physically plausible means of traveling between star systems is using generational colony ships driven only by some kind of sublight nuclear powered ion drive.

TuggyNE
2013-06-19, 01:33 AM
Strictly speaking, of course, the most physically plausible means of traveling between star systems is using generational colony ships driven only by some kind of sublight nuclear powered ion drive.

Are generation ships really more plausible than, say, cryonics?

factotum
2013-06-19, 02:22 AM
I agree with the majority opinion--there really isn't any "hard" science way of doing FTL travel. The Alcubierre drive requires things to exist that we don't know if they even *can* exist, which to my mind makes it about as hard scientifically as making someone disappear by waving a magic wand!

What you need to do is avoid technobabble and just have the FTL drive be there--such an integrated example of physics that nobody needs to sit down and explain it to anyone.

The Succubus
2013-06-19, 03:17 AM
Are generation ships really more plausible than, say, cryonics?

I think Rama had a good take on it. A miniturised world with a few rocket boosters strapped to it and flung out into the infinite depths of space. When it passes through an alien solar system, it gives them a few clues as to what life is like on an alien world, it's contained so as to minimise the risks of contamination and unlike a massive data archive, it doesn't need translating - they can just examine it.

Maryring
2013-06-19, 04:10 AM
It's not just about explaining it (or not) with technobabble in the story. I need to know for myself how it's going to work and try to ensure as strong an internal consistency as possible. Hence I need a solution, and one I can feel comfortable explaining and displaying should the need for it arise.

'sides. Can only help to have a look at the ideas and thoughts on the subject of other people. What I was toying with was quantum teleport/creating wormholes but I felt it might be a bit much. Exotic matter is not a problem though. I'm already assuming that to exist.

factotum
2013-06-19, 06:25 AM
I think Rama had a good take on it. A miniturised world with a few rocket boosters strapped to it and flung out into the infinite depths of space. When it passes through an alien solar system, it gives them a few clues as to what life is like on an alien world, it's contained so as to minimise the risks of contamination and unlike a massive data archive, it doesn't need translating - they can just examine it.

I don't think that was Rama's purpose, though, at least not in the original novel (we'll draw a discreet curtain over the Gentry Lee efforts)...it just passed through the solar system because it needed to make a course change and it needed energy from the sun to do it. It certainly wasn't ever intended as some sort of record for alien species to study!

razark
2013-06-19, 08:25 AM
Are generation ships really more plausible than, say, cryonics?
Yes. We have demonstrated the ability of the species to reproduce. We're still working on reliably freezing and thawing people while keeping them alive.


I need to know for myself how it's going to work and try to ensure as strong an internal consistency as possible.
Write out a set of rules for yourself on how it works. Not what makes it work, but what the effects/requirements of it are. Stick to those rules, even if you never actually refer to them. Even if it is not "hard", you can still make it consistent without directly exposing the reader to the technology used.

shawnhcorey
2013-06-19, 09:46 AM
Strictly speaking, of course, the most physically plausible means of traveling between star systems is using generational colony ships driven only by some kind of sublight nuclear powered ion drive.

The most physically plausible is to go really, really, fast. As the spaceship goes faster, time on board slows down (http://en.wikipedia.org/wiki/Twin_paradox). This takes a lot of energy but works.

Weezer
2013-06-19, 09:56 AM
Wormholes (more properly Einstein-Rosen Bridges) are definitively hard, so long as you deal with the requirements of using them. Namely exotic matter and white holes, two things that are theoretically possible, but have never been observed.

Ravens_cry
2013-06-19, 11:01 AM
Wormholes (more properly Einstein-Rosen Bridges) are definitively hard, so long as you deal with the requirements of using them. Namely exotic matter and white holes, two things that are theoretically possible, but have never been observed.
If you want to go harder still, they could be only used for communication, as the energy requirements for sending a series of laser pulses would be far smaller than anything of actual matter.

TuggyNE
2013-06-19, 07:32 PM
Yes. We have demonstrated the ability of the species to reproduce. We're still working on reliably freezing and thawing people while keeping them alive.

Ah, but have we demonstrated a reliable means of packing an ecosystem into a box and shipping it express to Alpha Centauri, C/O: distant descendants?

No, no we have not. Making a generation ship work requires solving a lot of Hard problems in ecosystem balance, maintaining healthy gene pools with comparatively small populations, making sure there's enough energy in various forms to keep things running, keeping the culture sane, and so on.

Ravens_cry
2013-06-19, 07:57 PM
It's obviously possible though. After all, we live in a system that is mostly closed ecosystem with the exception of energy input.

razark
2013-06-19, 08:07 PM
Ah, but have we demonstrated a reliable means of packing an ecosystem into a box and shipping it express to Alpha Centauri, C/O: distant descendants?
We're a lot further down that road than the freeze/thaw thing.


No, no we have not. Making a generation ship work requires solving a lot of Hard problems in ecosystem balance, maintaining healthy gene pools with comparatively small populations, making sure there's enough energy in various forms to keep things running, keeping the culture sane, and so on.
The question was "Is a generational ship more plausible?", not "Can we do it?". So I stand by my answer.

However, yes, we do have the technology to do it today. It would be expensive, both in terms of money, materials, energy, and lives; and there's no way anybody would actually do it. But it could be done, technically. (If you wanted to bankrupt the entire world.) There would be a lot of waste, as we do not understand everything about hand-maintaining an ecosystem, so we would need to engineer a ship with a massive "backup system" of extra capacity.

shawnhcorey
2013-06-19, 08:09 PM
It's obviously possible though. After all, we live in a system that is mostly closed ecosystem with the exception of energy input.

And its size gives it a buffering effect that guards us against messing it up too much, or a less it did until recently. A spaceship is a lot more fragile than a planet. It may not be possible to create a generation ship because it may not be stable long enough for none but the first few generations.

noparlpf
2013-06-19, 08:15 PM
Wouldn't it be easier to send frozen germ cells, which should be much easier to freeze and thaw than entire animals, and have them fertilised and grown once the ship gets where it's going? Have the ship set up camp, so to speak, and then raise the first generation. Two biggest issues: Nobody's gotten around to inventing the artificial womb (I'm picturing the classic movie cloning tube though) yet last I heard. And I'm terrified of AI (which is also a little bit off) so that's also a bad idea.

warty goblin
2013-06-19, 08:20 PM
Ah, but have we demonstrated a reliable means of packing an ecosystem into a box and shipping it express to Alpha Centauri, C/O: distant descendants?

No, no we have not. Making a generation ship work requires solving a lot of Hard problems in ecosystem balance, maintaining healthy gene pools with comparatively small populations, making sure there's enough energy in various forms to keep things running, keeping the culture sane, and so on.

I think you'd need to solve the ecosystem in a jar problem anyway, unless you get extremely lucky with your destination planet. It seems a bit chancy to haul ass across space without having a secure means of growing food or ensuring breathable air upon arrival.

The major advantage of the homo icecube approach is that when your ship shows up three or ten generations later, it's not full of a bunch of people with the most hardcore cases of agoraphobia hitherto experienced in the species. A couple hundred Gollums in a tin can does not seem a good choice for the foundation of a new civilization, does it Precious?

noparlpf
2013-06-19, 08:29 PM
Why are we picturing a spaceship? THINK BIG. Commandeer a large moon. Terraform it. Stick on some form of propulsion.

Flickerdart
2013-06-19, 08:34 PM
The hardest way to do FTL would be to keep the ship perfectly still and move the entire universe around it.

warty goblin
2013-06-19, 08:40 PM
Why are we picturing a spaceship? THINK BIG. Commandeer a large moon. Terraform it. Stick on some form of propulsion.

I think this ends up even harder to pull off in a lot of ways. For one thing you can't just keep your atmosphere contained anymore, unless you build a big shell over the entire planetoid. For another, propelling a largish planetoid is a very non-trivial undertaking, to the point where I'd be seriously worried about baking the whole thing with residual heat from my engines. At the same time heating the thing gets hard when you aren't firing the engines , and the ridiculous fuel requirements of a mobile planet ensure you want to leave them off as much as possible. Space doesn't wick away heat very fast, but there's a lot of it between stars, and many years to bleed your thermal energy away.

So without some very unlikely sorts of technology, your flight would be something like a short, extremely hot period in which your atmosphere ignites and literally burns off into space, followed by a long stretch where everything gets progressively colder and colder until Antarctica in July looks like a day at the beach, followed by another brief inferno period as you slow your planet down at the other end.

shawnhcorey
2013-06-19, 08:46 PM
The hardest way to do FTL would be to keep the ship perfectly still and move the entire universe around it.

The hardest way would be to shrink the universe to a very small size, move a little distant in it, and re-expand it, thus discovering that you are very, very far from your start.

Weezer
2013-06-19, 09:02 PM
If you want to go harder still, they could be only used for communication, as the energy requirements for sending a series of laser pulses would be far smaller than anything of actual matter.

And that would deal with the sticky problem of needing to make a ship that could survive the gravitation of a wormhole.

noparlpf
2013-06-19, 09:24 PM
I think this ends up even harder to pull off in a lot of ways. For one thing you can't just keep your atmosphere contained anymore, unless you build a big shell over the entire planetoid. For another, propelling a largish planetoid is a very non-trivial undertaking, to the point where I'd be seriously worried about baking the whole thing with residual heat from my engines. At the same time heating the thing gets hard when you aren't firing the engines , and the ridiculous fuel requirements of a mobile planet ensure you want to leave them off as much as possible. Space doesn't wick away heat very fast, but there's a lot of it between stars, and many years to bleed your thermal energy away.

So without some very unlikely sorts of technology, your flight would be something like a short, extremely hot period in which your atmosphere ignites and literally burns off into space, followed by a long stretch where everything gets progressively colder and colder until Antarctica in July looks like a day at the beach, followed by another brief inferno period as you slow your planet down at the other end.

Solar sails. No engine heat, and you move pretty slowly to start; once you leave the system, you'll just coast. If you're not worried about getting there, just having the planetoid and your distant offspring get there, speed is no issue.


The hardest way to do FTL would be to keep the ship perfectly still and move the entire universe around it.


The hardest way would be to shrink the universe to a very small size, move a little distant in it, and re-expand it, thus discovering that you are very, very far from your start.

You guys are thinking way too high-tech. The hardest way to reach FTL speeds would be to try to run there. Your ways, somebody could come up with some mathematical model of it. My way, people will just laugh in my face and wish me luck.

Palanan
2013-06-19, 09:31 PM
It seems to me that before we get any further into generation ships and such--to say nothing of terraformed moons--it would be helpful if Maryring could tell us a little more about what she wants her particular starship to do.

Apart from "big," I don't see too many other parameters, and "big" is purely relative. The Cassini probe is "big" compared to Sputnik; and then there's the dragon-ship from The Copper Crown, which is ten miles long; and then we get into the genuine world-ships, of which hollowed asteroids are only the beginning.

I will say to the OP that if she wants to be rigorous ("hard") then mass is a real consideration, and a generation ship is probably the most inefficient use of mass out there. In Lovelock, which was a short-lived collaboration between Orson Scott Card and Kathryn Kidd, the colonists selected for a quasi-generation ship had a mass allowance of a thousand pounds per person, which always struck me as absurd. Apart from the social and genetic issues mentioned above, a generation ship is just a terrible way to do business.

But again, it depends on what Maryring wants to do with this ship. Colonization, exploration, stargate pusher, Van Neumann seed?

.

Hiro Protagonest
2013-06-19, 09:38 PM
Yeah, I think pretty much the easiest way to get FTL is to make a warp drive that works by temporarily creating a wormhole.

Also, in an hour-long Phineas and Ferb special where they chase the sun on the Summer solstice, they made a giant paper plane out of a giant map to get the rest of the way home. And while Candace said "you know that folding the paper doesn't make these places closer together?", that's a fairly plausible explanation for why it worked. :smallamused: The other one, of course, being that they may have landed on an island in the Bermuda Triangle. Although that would be pretty far south of their intended destination of New York. :smallconfused:


The hardest way to do FTL would be to keep the ship perfectly still and move the entire universe around it.

Nah, just make sure everyone on board is a narcissist.

Ravens_cry
2013-06-19, 10:55 PM
Yeah, I think pretty much the easiest way to get FTL is to make a warp drive that works by temporarily creating a wormhole.

Not really. You would need an open 'throat' at your destination all ready to get there. A FTL idea I had for an otherwise fairly hard sci-fi universe was a metric jump. Basically, you transfer over to another, smaller 'brane, travel along it at slower than light, and then jump back, having gone proportionally farther.

And that would deal with the sticky problem of needing to make a ship that could survive the gravitation of a wormhole.
A black hole, which is *one way* of making a wormhole, only has the gravitation of the mass it was made from. So a sufficinently low mass black hole won't cause any more problems than the ship itself. Of course, then you have to worry about it dissolving in a spray of Hawking radiation, but any mass, compressed enough, becomes a blackhole.

Flickerdart
2013-06-19, 10:59 PM
You guys are thinking way too high-tech. The hardest way to reach FTL speeds would be to try to run there. Your ways, somebody could come up with some mathematical model of it. My way, people will just laugh in my face and wish me luck.
Too easy, someone might be the Flash. Real hardest way is to sit down and wish really hard.

noparlpf
2013-06-19, 11:12 PM
Too easy, someone might be the Flash. Real hardest way is to sit down and wish really hard.

Wishing is easy, even if it gets you nowhere fast. But running with the serious hope that you'll reach FTL speeds? Exhausting, and emotionally devastating when you finally realise (the fairly obvious fact) that the human body can't do it.
And anyway, not a DC fan, but can the various Flashes even reach light speed? Even if they can, could they run at light speed through the vacuum of space, where there's nothing to run on?

Ravens_cry
2013-06-19, 11:19 PM
He once beat Death by running fast enough that the time dilation meant that all other life in the universe was extinct at that point. So, yes, the Flash can run in space. . . and get somewhere.

noparlpf
2013-06-19, 11:21 PM
He once beat Death by running fast enough that the time dilation meant that all other life in the universe was extinct at that point. So, yes, the Flash can run in space. . . and get somewhere.

I suppose the magical aura that protects them from air resistance also protects them from lack of friction?

factotum
2013-06-20, 02:12 AM
And anyway, not a DC fan, but can the various Flashes even reach light speed? Even if they can, could they run at light speed through the vacuum of space, where there's nothing to run on?

Technically, nothing that has mass can actually *reach* lightspeed, just get arbitrarily close to it...

shawnhcorey
2013-06-20, 06:27 AM
A black hole, which is *one way* of making a wormhole, only has the gravitation of the mass it was made from. So a sufficinently low mass black hole won't cause any more problems than the ship itself. Of course, then you have to worry about it dissolving in a spray of Hawking radiation, but any mass, compressed enough, becomes a blackhole.

Sorry, no. All black holes have infinite gravity at their event horizon. Hawking radiation is insignificant compared to the radiation emitted by the material that falls down it and there is always enough to create large amounts of radiation. And the other problem are the tidal forces that will pull your ship apart. If they get close enough, even hadrons will be pull apart.

Frozen_Feet
2013-06-20, 06:45 AM
Yes. We have demonstrated the ability of the species to reproduce.

Actually, we have demonstrated the opposite in space. All earthbound life save for microbes is screwed over by zero-gravity environment. Embryos fail to develop correctly, bones become brittle, muscles athropy... actually giving birth on a spaceship looks like a recipe of certain disaster in light of current knowledge.

A generational ship might not survive beyond the first generation, and even if it did, the following generations would be unfit for life in a gravity well.

MikelaC1
2013-06-20, 06:49 AM
I think what you need to look at is what type of stories you intend to write. If you are not writing stories about how the ships travel and just why they travel and what they do, you can just hand wave the FTL speed explanation away. As has been pointed out, people are not always going to discuss how an internal combustion engine works in a car as they drive to work, they are going to discuss current events. If your stories are going to focus on that, then leave the mechanics of the FTL in the background...they just can do it, end of discussion.

Vortalism
2013-06-20, 06:50 AM
If you wanna go really really hard on Moh's scale try the Wikipedia article it's got a variety of very complicated theoretical physics and thought experiments on faster-than-light travel from the warp (alcubierre) drive example - (basically moving the universe around the ship) to a krasnikov tube to some really bizarre stuff like Heim theory (using magnetic fields to enter high dimensional space) to some really bizarre stuff.. Have fun. :smallbiggrin:

Or better yet, as addressed before, don't try to.

razark
2013-06-20, 08:55 AM
All earthbound life save for microbes is screwed over by zero-gravity environment.
So we get rid of the 0g environment. We've constructed a giant world-ship, capable of propelling itself to distant stars, carrying more than enough to sustain multiple generations of humans.

Spinning that sucker up to create a 1g effect is going to be rather trivial by comparison.

shawnhcorey
2013-06-20, 09:08 AM
...carrying more than enough to sustain multiple generations of humans.

Ah, but that's the trick. We're not sure that is possible. The ship would have to be really, really big to ensure that minor disasters aren't major ones.

noparlpf
2013-06-20, 09:25 AM
Actually, we have demonstrated the opposite in space. All earthbound life save for microbes is screwed over by zero-gravity environment. Embryos fail to develop correctly, bones become brittle, muscles athropy... actually giving birth on a spaceship looks like a recipe of certain disaster in light of current knowledge.

A generational ship might not survive beyond the first generation, and even if it did, the following generations would be unfit for life in a gravity well.

It's not all that hard to come up with artificial "gravity" on a ship. Just set it spinning so the outer levels experience about one gee.

razark
2013-06-20, 09:52 AM
The ship would have to be really, really big to ensure that minor disasters aren't major ones.
Uh, yeah. Covered that:

There would be a lot of waste, as we do not understand everything about hand-maintaining an ecosystem, so we would need to engineer a ship with a massive "backup system" of extra capacity.
To be clear, the sort of ship involved is the sort that generates comments like "That's no moon.".

It is doable. The problem is that it is way too expensive to build. Expensive to the point of needing to strip mine the earth, the moon, and maybe Mars to build it. We would need such a ship if we built one, because the earth would be left an uninhabitable wasteland.

hamishspence
2013-06-20, 09:56 AM
Could it not be done by using asteroids? Or are there not enough resources in the Belt to put together a good vessel?

Animastryfe
2013-06-20, 10:09 AM
The Orion's Arm project is " an online collaborative fiction setting which describes itself as a transhumanist Space Opera with many worldbuilding elements"[1] (http://tvtropes.org/pmwiki/pmwiki.php/Main/OrionsArm). Generally, Orion's Arm does not use any physics that has been proven to be impossible using modern theories, although many of the physics that it uses is speculative due to the nature of the project. In this setting, true FTL travel has not been discovered, but people use traversible Lorentzian wormholes (http://www.orionsarm.com/eg-article/48545a0f6352a) to travel in a way that is not true FTL, but drastically minimizes travel time. Generally, Orion's Arm does not use any physics that has been proven to be impossible using modern theories

Emmerask
2013-06-20, 10:15 AM
Asteroid mining has a ton of upsides and some downsides that need to be addressed and overcome.

There is more then enough of the material needed

In fact, all the gold, cobalt, iron, manganese, molybdenum, nickel, osmium, palladium, platinum, rhenium, rhodium, ruthenium, and tungsten mined from Earth's crust, and that are essential for economic and technological progress, came originally from the rain of asteroids that hit Earth after the crust cooled

However mining in essentially zero g is very hard, you cant just use explosives for example... the material would just fly away.

It would need to be completely automated, everything else would just cost to many resources.

And asteroid belts are not really a safe environment for a ship I would think.

http://en.wikipedia.org/wiki/Asteroid_mining for more information

but overall I think this is a much better approach then to mine planets.

Frozen_Feet
2013-06-20, 10:19 AM
So we get rid of the 0g environment. We've constructed a giant world-ship, capable of propelling itself to distant stars, carrying more than enough to sustain multiple generations of humans.


That's called a planet. If you can create a planet, what is your incentive to travelling to other worlds? Also, propelling a planet anywhere takes ridiculous amounts of energy. (It should be noted that our solar system is hardly fixed in place - it is moving around the Milkyway with quite startling speed, but I'm not aware if this movement puts us in course with any other solar systems in timespans relatable to human civilization.


It's not all that hard to come up with artificial "gravity" on a ship. Just set it spinning so the outer levels experience about one gee.

According to what I've read, it's iffy whether that actually counteracts the effect. Yes, conservation of momentum and coriolis effect simulate being "pulled" into a surface, but does it affect biological cells in the same way as actual gravity is unsolved.

razark
2013-06-20, 10:19 AM
Could it not be done by using asteroids? Or are there not enough resources in the Belt to put together a good vessel?
Yes, we could possibly mine the asteroids. But first we need to build a massive fleet of ships to go mine them.


If you can create a planet, what is your incentive to travelling to other worlds?
I never said it was a good idea or it's something we should do. Just that it is technically possible.

noparlpf
2013-06-20, 10:25 AM
According to what I've read, it's iffy whether that actually counteracts the effect. Yes, conservation of momentum and coriolis effect simulate being "pulled" into a surface, but does it affect biological cells in the same way as actual gravity is unsolved.

...Isn't force force? Why should it matter what's causing it as long as it's uniform and constant?

Weezer
2013-06-20, 10:25 AM
Yes, we could possibly mine the asteroids. But first we need to build a massive fleet of ships to go mine them.



Nah, just build a few and use some of the stuff they mine to make more. After they're done with all the mining recycle them and incorporate them into the final ship.

Frozen_Feet
2013-06-20, 10:38 AM
...Isn't force force? Why should it matter what's causing it as long as it's uniform and constant?

Really, it came as a suprise how strongly affected development of organisms is by gravity is. Plant seeds apparently have some sort of gravitic sense to help them grow. It's not a fully understood phenomenom.

Putting iron dust inside a whirling ring will cause the dust to stick to the interior of the ring, but not in the same way as it would if the ring was strongly electromagnetic. The type of force could matter, but whether it does and how is not solved.

Speaking of which, even a human body has cells that are reactive to magnetic field of Earth. If a ship has different magnetic field, it potentially has consequences for biological life forms as well.

noparlpf
2013-06-20, 10:43 AM
Really, it came as a suprise how strongly affected development of organisms is by gravity is. Plant seeds apparently have some sort of gravitic sense to help them grow. It's not a fully understood phenomenom.

Putting iron dust inside a whirling ring will cause the dust to stick to the interior of the ring, but not in the same way as it would if the ring was strongly electromagnetic. The type of force could matter, but whether it does and how is not solved.

Speaking of which, even a human body has cells that are reactive to magnetic field of Earth. If a ship has different magnetic field, it potentially has consequences for biological life forms as well.

Well, for plants, I know that a downwards force helps affect local concentrations of hormones, and cells respond differently to different concentrations or gradients of hormones. I'm fairly certain that gravitational and centrifugal forces would have the same effect.
I hadn't thought of the Earth's magnetic field, but presumably it would be trivial to simulate, considering how weak it is.

shawnhcorey
2013-06-20, 10:45 AM
...Isn't force force? Why should it matter what's causing it as long as it's uniform and constant?

But neither gravity or centrifugal forces are uniform. The non-uniformity of gravity causes tides; the non-uniformity of centrifugal forces cause Coriolis forces.

Ravens_cry
2013-06-20, 10:50 AM
Sorry, no. All black holes have infinite gravity at their event horizon. Hawking radiation is insignificant compared to the radiation emitted by the material that falls down it and there is always enough to create large amounts of radiation. And the other problem are the tidal forces that will pull your ship apart. If they get close enough, even hadrons will be pull apart.
Which again is solvable by making the blackhole low mass enough, and therefore giving it a small enough event horizon .
Put it this way, if the Sun was somehow transformed into a blackhole, the Earth would continue to orbit it just fine.

noparlpf
2013-06-20, 11:10 AM
But neither gravity or centrifugal forces are uniform. The non-uniformity of gravity causes tides; the non-uniformity of centrifugal forces cause Coriolis forces.

In a simple hypothetical system, gravity is uniform at a given distance from a point mass. What I meant was, gravity at the Earth's surface is approximately uniform, because the Earth's mass is roughly evenly distributed, and its radius is approximately equal at most points. Similarly, for a wheel-shaped ship, centrifugal force should be approximately uniform on a given level because the floor is at a constant distance from the center of rotation.

As I understand it, tides are primarily caused by the gravitational forces of the Sun and Moon, not lack of uniformity of Earth's gravity. A ship is typically out in the middle of nowhere, so gravitational effects from stars and other bodies should be negligible in between solar systems.

shawnhcorey
2013-06-20, 11:28 AM
Which again is solvable by making the blackhole low mass enough, and therefore giving it a small enough event horizon .
Put it this way, if the Sun was somehow transformed into a blackhole, the Earth would continue to orbit it just fine.

You mean if the ship did not go close enough to a black hole so that it is not greatly effected by it. Even low-mass black holes has infinite gravity at their event horizons.

And the Earth would be effected if the Sun became a black hole. Assuming that a distributed body has the same gravity as a point source only works when the body is far away. When it is close, you have to that into account the distribution of the mass. These extra forces are called tidal forces.


In a simple hypothetical system, gravity is uniform at a given distance from a point mass. What I meant was, gravity at the Earth's surface is approximately uniform, because the Earth's mass is roughly evenly distributed, and its radius is approximately equal at most points. Similarly, for a wheel-shaped ship, centrifugal force should be approximately uniform on a given level because the floor is at a constant distance from the center of rotation.

You mean is its diameter is large enough so it could create a 1g centrifugal force with a slow rotation, then the Coriolis forces would be small enough to ignore.


As I understand it, tides are primarily caused by the gravitational forces of the Sun and Moon, not lack of uniformity of Earth's gravity. A ship is typically out in the middle of nowhere, so gravitational effects from stars and other bodies should be negligible in between solar systems.

The tides are caused by the difference in gravity from the Moon and Sun from one side of the Earth to the other. Relative to the force from the Moon experienced at the centre of the Earth, the Moon's gravity creates forces pushing outward on the side closest to the Moon and farthest from it and other forces that push inward at the circumference around the horizon of the Moon.

Ravens_cry
2013-06-20, 12:05 PM
You mean if the ship did not go close enough to a black hole so that it is not greatly effected by it. Even low-mass black holes has infinite gravity at their event horizons.

And the Earth would be effected if the Sun became a black hole. Assuming that a distributed body has the same gravity as a point source only works when the body is far away. When it is close, you have to that into account the distribution of the mass. These extra forces are called tidal forces.
Moon.
Abnd what counts as 'far away' depends on the mass of the blackhole. For one the mass of the sun, the Earth is more than far away enough to not worry about the tidal effects. For one low mass enough, and a ship big enough, you just need to keep the black hole far away enough from everything else. Of course, there is more than one way to make a wormhole, though negative energy, with the sort of exception of the Casimir effect, is still basically hypothetical.

hamishspence
2013-06-20, 12:09 PM
Yes, we could possibly mine the asteroids. But first we need to build a massive fleet of ships to go mine them.

While the writing style leaves something to be desired, John Ringo's Live Free or Die series, had an interesting "quick and dirty" means of producing a 9 km wide spaceship.

Step one- find nickel-iron asteroid.
Step two- find comet
Step three- drill hole in asteroid
Step four- insert comet
Step five- melt hole shut with sunbeam weapon
Step six- heat asteroid with sunbeam weapon.

The comet inside, due to the heat, turns to gas, and blows the asteroid up like a balloon.

The idea apparently came from much older sci-fi.

The sunbeam weapon consists of a whole lot of mirrors, concentrating the light of the sun into something hot enough to melt things easily.

shawnhcorey
2013-06-20, 12:21 PM
Abnd what counts as 'far away' depends on the mass of the blackhole. For one the mass of the sun, the Earth is more than far away enough to not worry about the tidal effects. For one low mass enough, and a ship big enough, you just need to keep the black hole far away enough from everything else. Of course, there is more than one way to make a wormhole, though negative energy, with the sort of exception of the Casimir effect, is still basically hypothetical.

The solar tide on Earth is about 1/3 that of the Moon's tide. I would say it's close enough to be significant.

The Casimir effect is about decreasing vacuum energy. If you assume vacuum have no energy to start with, then it would create negative energy. But many now assume that vacuum still has energy. Some estimates (http://en.wikipedia.org/wiki/Vacuum_energy) put it at 10^113 Joules per cubic meter, an incredible amount.

Soras Teva Gee
2013-06-20, 12:27 PM
But neither gravity or centrifugal forces are uniform. The non-uniformity of gravity causes tides; the non-uniformity of centrifugal forces cause Coriolis forces.

Earth's gravity can be considered constant though. Actually it well must be that's how gravity works. And its the dominant force in the system. Now okay your at a different spot with the tides the moon and sun and all you add more vectors to that system, but Earth's gravity remains constant there's simply more forces at work.

And since this came up in the context of living beings and physiology your going to have to get some actual verification to establish that the force is enough to actually do anything in a meaningful way.

I mean heck we're technically being pulled everywhere else in the universe by gravity.

Mando Knight
2013-06-20, 12:52 PM
Which again is solvable by making the blackhole low mass enough, and therefore giving it a small enough event horizon .
Put it this way, if the Sun was somehow transformed into a blackhole, the Earth would continue to orbit it just fine.

Yes, but you're trying to go through the black hole. Which by necessity exposes your vessel to the extreme gravitational stresses of approaching the event horizon.

shawnhcorey
2013-06-20, 01:16 PM
Earth's gravity can be considered constant though. Actually it well must be that's how gravity works. And its the dominant force in the system. Now okay your at a different spot with the tides the moon and sun and all you add more vectors to that system, but Earth's gravity remains constant there's simply more forces at work.

Earth's gravity is not constant even on its surface. Go around to the other side of the planet and you'll see that it is pulling in the opposite direction. Earth's gravity is radially inward.

noparlpf
2013-06-20, 01:25 PM
You mean is its diameter is large enough so it could create a 1g centrifugal force with a slow rotation, then the Coriolis forces would be small enough to ignore.

Could you explain to me how Coriolis forces would affect things on one level of this wheel? I'm picturing a large wheel a couple of kilometers thick.

Eldan
2013-06-20, 01:30 PM
Asteroid belts really aren't all that dangerous.
NASA had a nice quote. They are happy if they even see an asteroid from their probes when flying through the belt.

hamishspence
2013-06-20, 01:40 PM
There are denser belts in other systems though.

Soras Teva Gee
2013-06-20, 01:43 PM
There are denser belts in other systems though.

That's not saying much

Anything as half as dense as what you see in the movies would bash itself to dust in next to no time.

You'd need some kind of recent catastrophe (maybe) not an endemic natural feature.

hamishspence
2013-06-20, 01:46 PM
Or a star system that's right at the beginning of planetary formation- thus, belt is a lot more dust than rocks.

shawnhcorey
2013-06-20, 01:51 PM
Could you explain to me how Coriolis forces would affect things on one level of this wheel? I'm picturing a large wheel a couple of kilometers thick.

At that size, the Coriolis forces would be small. Pop-flies and golf balls would do strange curves though.


There are denser belts in other systems though.

Saturn's Rings are an example of this.

Gravitron5000
2013-06-20, 02:08 PM
In a simple hypothetical system, gravity is uniform at a given distance from a point mass. What I meant was, gravity at the Earth's surface is approximately uniform, because the Earth's mass is roughly evenly distributed, and its radius is approximately equal at most points. Similarly, for a wheel-shaped ship, centrifugal force should be approximately uniform on a given level because the floor is at a constant distance from the center of rotation.

The floor is a constant difference from the center of rotation, but given that people are around 1.5 m tall, depending on the size of your ship, there could still be a noticeable difference in gravity between your head and your toes. 1.5 m is negligible when compared to the 6.4 million meters we are from the center of the earth, but if you are rotating around an axis at 50 m, you'll probably notice the gradient, especially if you decide to jump.

noparlpf
2013-06-20, 02:15 PM
At that size, the Coriolis forces would be small. Pop-flies and golf balls would do strange curves though.

I'm not sure how many projectile ball games people would think to bring on a spaceship.
Anyway, a generation ship doesn't have to be all that big, so it would probably have to spin fairly quickly to maintain a gee, but I don't think Coriolis forces would drastically affect the humans or animals living on the ship.

Edit:

The floor is a constant difference from the center of rotation, but given that people are around 1.5 m tall, depending on the size of your ship, there could still be a noticeable difference in gravity between your head and your toes. 1.5 m is negligible when compared to the 6.4 million meters we are from the center of the earth, but if you are rotating around an axis at 50 m, you'll probably notice the gradient, especially if you decide to jump.

A wheel 100m across is pretty small, though. Assuming a generation ship includes up to maybe ten thousand humans alive at a time, and a bunch of animals of various sorts, you'd need probably around 100-200km˛ (which is generous, considering some small countries with less area have more people), which would probably require a radius of around 15-20km. I think 1.5m is pretty much negligible at that size. Note that all of these numbers are off the top of my head, so anybody who wants to do real math is welcome to.

Palanan
2013-06-20, 04:05 PM
Originally Posted by hamishspence
Step one- find nickel-iron asteroid.
Step two- find comet
Step three- drill hole in asteroid
Step four- insert comet
Step five- melt hole shut with sunbeam weapon
Step six- heat asteroid with sunbeam weapon.

This approach appeared in a novel by Larry Niven from the 60s or 70s, although he might have borrowed it from older sources in turn. In his novel (can't recall which one at the moment) the expanded asteroid was used as a moderate-g environment for birthing mothers.

VeisuItaTyhjyys
2013-06-20, 04:52 PM
For example, when you read a contemporary novel, the author does not explain the physics of flight when a character rides in a plane. Nor does an author explain the intricacies of an internal combustion engine when a character drives a car.

Hey. Hey. The author does not necessarily explain the physics of flight or the intricacies of an internal combustion engine. Some of us have a weird Herman Melville/Tom Pynchon thing going on. :smalltongue:


There's also the old standbye of wormholes/fold drives if you don't fuss on the details and energy requirements too much. Since its exploiting a principle that nominally exists in science.

"Energy requirements? We just bring a lot of crates." :smallwink:

Soras Teva Gee
2013-06-20, 05:49 PM
I'm not sure how many projectile ball games people would think to bring on a spaceship.
Anyway, a generation ship doesn't have to be all that big, so it would probably have to spin fairly quickly to maintain a gee, but I don't think Coriolis forces would drastically affect the humans or animals living on the ship.


The O'Neill Cylinder Island 3 style proposal would measure 8 km diameter and 32 km long. And its nominally intended for indefinite habitation so a closed variation would probably be a ballpark figure to start with. And at 8 km in diameter variations under 1% probably I think would have to be below perception.

Of course you'd probably want a few of these in a fleet for redundancy if nothing else.

The more interesting question would be power and providing a light source when away from anything as convenient as the sun.

the_druid_droid
2013-06-20, 07:04 PM
Ayup. Comparing the hardness of FTL methods in sci-fi is rather like judging which type of gelatin best survives having cinder blocks dropped on it from great height: ultimately the differences are irrelevant to the big, sticky mess the enterprise makes out of everything. Just go with whatever flavor you find works best with your other ingredients.

Pretty much this. The vast majority (I'm tempted to say all, but I'm sure someone has a counter-example) of even the "legitimate" proposals hinge on things which a.) are included to make the equations work and b.) have zero experimental evidence to back them as actually existing.

As far as truly experimentally verified physics goes, light speed is the limit. Period.

(Yes, I'm ignoring tachyons, but that's really just the light speed problem turned on its head.)


Strictly speaking, of course, the most physically plausible means of traveling between star systems is using generational colony ships driven only by some kind of sublight nuclear powered ion drive.

I suppose it's been touched on already, but unless we're talking about doing something on a truly massive scale (which brings in even more issues), this runs into the laws of thermodynamics very hard and very quickly, at least generationally speaking.


'sides. Can only help to have a look at the ideas and thoughts on the subject of other people. What I was toying with was quantum teleport/creating wormholes but I felt it might be a bit much. Exotic matter is not a problem though. I'm already assuming that to exist.

Ok, if exotic matter exists, then some of the solutions from earlier are probably your best bet. I would advise against "quantum" anything though; all too often it's used interchangeably with "magic" by sci-fi writers, and you need double the caution if you're actually proposing quantum effects on a macroscopic scale like a portal would seem to suggest. Patching together the microscopic and macroscopic domains of quantum mechanics is still a very open question in physics, and there isn't really any way to make it "hard" in the sense I think you mean.


Why are we picturing a spaceship? THINK BIG. Commandeer a large moon. Terraform it. Stick on some form of propulsion.

That has an absolutely tremendous energy cost, even ignoring the terraforming aspect. This is mainly because the most effective way to move things of that size in space is gravity, and that puts us right back into the speculative physics boat, at least as far as generating that sort of gravity well goes.

Not to mention the energy required to get it out of whatever local gravity well you're in (presumably the solar one, to start off).


Solar sails. No engine heat, and you move pretty slowly to start; once you leave the system, you'll just coast. If you're not worried about getting there, just having the planetoid and your distant offspring get there, speed is no issue.

Solar sails really aren't viable at that scale. Their impulse is tiny, and really only works as a cumulative effect for small objects. Trying to move a planet(oid) with them is like trying to support a lead block with tissue paper, let alone the issue of how to attach them and deploy them in the first place.

noparlpf
2013-06-20, 07:41 PM
The O'Neill Cylinder Island 3 style proposal would measure 8 km diameter and 32 km long. And its nominally intended for indefinite habitation so a closed variation would probably be a ballpark figure to start with. And at 8 km in diameter variations under 1% probably I think would have to be below perception.

Of course you'd probably want a few of these in a fleet for redundancy if nothing else.

The more interesting question would be power and providing a light source when away from anything as convenient as the sun.

I've read about at least one other big cylinder like that too. I think it's a common design besides the wheel-with-spokes image.
Lighting is fairly easy. We have high-intensity and full-spectrum lights. Energy is the main issue when you're light-years from the nearest star. We would probably have to use some sort of fancy spacefuture nuclear power.

Ravens_cry
2013-06-20, 07:53 PM
The solar tide on Earth is about 1/3 that of the Moon's tide. I would say it's close enough to be significant.

The Casimir effect is about decreasing vacuum energy. If you assume vacuum have no energy to start with, then it would create negative energy. But many now assume that vacuum still has energy. Some estimates (http://en.wikipedia.org/wiki/Vacuum_energy) put it at 10^113 Joules per cubic meter, an incredible amount.
To create those kinds of tide, yes. But that's something it already does. The kind of tidal forces that create such colourful effects as dangerous spagetifaction (http://en.wikipedia.org/wiki/Spaghettification) require one to be much, much closer. Here, read this (http://www.badastronomy.com/bad/misc/black_hole_sun.html).

warty goblin
2013-06-20, 10:46 PM
Ok, if exotic matter exists, then some of the solutions from earlier are probably your best bet. I would advise against "quantum" anything though; all too often it's used interchangeably with "magic" by sci-fi writers, and you need double the caution if you're actually proposing quantum effects on a macroscopic scale like a portal would seem to suggest. Patching together the microscopic and macroscopic domains of quantum mechanics is still a very open question in physics, and there isn't really any way to make it "hard" in the sense I think you mean.

Meh. FTL is all magic space unicorns anyway. Caring about whether it's quantum or wormhole or extra dimension based is like getting kvetchy that somebody named their magic space unicorn Mr. Sparklemane instead of Prancytoes.


(Which suggests the solution of having your rockets pulled by actual magic space unicorns.)

Ravens_cry
2013-06-20, 11:28 PM
Yes, FTL looks either impossible or unattainable by real world physics, but that does not mean you can't make a self-consistent system that operates according to its own rules, with interesting look into some of the consequences of the technology.

VeisuItaTyhjyys
2013-06-21, 12:15 AM
Meh. FTL is all magic space unicorns anyway. Caring about whether it's quantum or wormhole or extra dimension based is like getting kvetchy that somebody named their magic space unicorn Mr. Sparklemane instead of Prancytoes.


(Which suggests the solution of having your rockets pulled by actual magic space unicorns.)

Magical Space Unicorns are my new favorite kind of exotic matter.

factotum
2013-06-21, 02:08 AM
Magical Space Unicorns are my new favorite kind of exotic matter.

And if you make them dead black as well, you have the solution for where all the dark matter physicists keep looking for is! :smallsmile:

Aotrs Commander
2013-06-21, 07:14 AM
And if you make them dead black as well, you have the solution for where all the dark matter physicists keep looking for is! :smallsmile:

There was, in fact, (in)famously, a suggestion that one of the characters in Friendship is Magic was made of Dark Matter, from a chap who chose the show to do a physics lecture on...!

Eldan
2013-06-21, 08:00 AM
I choose to believe that the Invisible Pink Unicorn is made of Dark Matter.

warty goblin
2013-06-21, 08:18 AM
Yes, FTL looks either impossible or unattainable by real world physics, but that does not mean you can't make a self-consistent system that operates according to its own rules, with interesting look into some of the consequences of the technology.


Self-consistent as long as whenever somebody mentions causality everybody else hums really loudly.

All of the really interesting things about looking into the effects of the technology can be done with very little technobabble. Call your FTL magic space unicorns, and suddenly places where gamma radiation grass grows become vitally important. Mr. Sparklemane can't run all the way from Earth to Epsilon Beta Delta 7 without some R&R after all. Call it quantum jumpgates, and jumpgate locations are the new hot deal.

Ravens_cry
2013-06-21, 11:13 AM
Self-consistent as long as whenever somebody mentions causality everybody else hums really loudly.

Well, only if relativity also works in that world. Also, I have read a few storie where FTL basically can equal time travel to the past was exploited as part of the plot and story.


All of the really interesting things about looking into the effects of the technology can be done with very little technobabble. Call your FTL magic space unicorns, and suddenly places where gamma radiation grass grows become vitally important. Mr. Sparklemane can't run all the way from Earth to Epsilon Beta Delta 7 without some R&R after all. Call it quantum jumpgates, and jumpgate locations are the new hot deal.
Something like that, I suppose.

Deepbluediver
2013-06-21, 11:31 AM
The major advantage of the homo icecube approach is that when your ship shows up three or ten generations later, it's not full of a bunch of people with the most hardcore cases of agoraphobia hitherto experienced in the species. A couple hundred Gollums in a tin can does not seem a good choice for the foundation of a new civilization, does it Precious?

What I'm hearing is that our generation-ship will need to be crewed by the Amish, which is already an, insular, peacful, highly-supportive and low-energy society, which has all the skills (farming, crafting, building) to colonize a new world. :smallwink:

And given that their entire schtick is divorcing themselves from the troubles of the world, you probably wounldn't even have to try that hard to convince them to give it a go.

Ronnoc
2013-06-21, 12:03 PM
Sorry, no. All black holes have infinite gravity at their event horizon. Hawking radiation is insignificant compared to the radiation emitted by the material that falls down it and there is always enough to create large amounts of radiation. And the other problem are the tidal forces that will pull your ship apart. If they get close enough, even hadrons will be pull apart.

Actually the tidal forces at the event horizon are large but finite and inversely proportional to the mass of the black hole. For a black hole with a mass of approximately 10^9 solar masses you wouldn't even notice the tidal forces at the event horizon. That said the tidal forces would still approach infinity as you approach the center of the black hole some finite amount of time later.

That said you are correct in that Hawking radiation is insignificant compared to radiation generated by accelerating matter in the accretion disk and magnetic interactions. The only case in which hawking radiation is likely to be measurable is when a black hole evaporates, which do to the time scales involved would only be seen in the smallest of black holes.

shawnhcorey
2013-06-21, 12:19 PM
Actually the tidal forces at the event horizon are large but finite and inversely proportional to the mass of the black hole. For a black hole with a mass of approximately 10^9 solar masses you wouldn't even notice the tidal forces at the event horizon. That said the tidal forces would still approach infinity as you approach the center of the black hole some finite amount of time later.

No, nothing can approach the centre of a black hole. It can only approach its event horizon. And gravity at event horizon is infinite.


That said you are correct in that Hawking radiation is insignificant compared to radiation generated by accelerating matter in the accretion disk and magnetic interactions. The only case in which hawking radiation is likely to be measurable is when a black hole evaporates, which do to the time scales involved would only be seen in the smallest of black holes.

No, Hawking's radiation has already been measured because it has characteristics that are different that those created by accelerating matter.

hamishspence
2013-06-21, 12:37 PM
No, nothing can approach the centre of a black hole. It can only approach its event horizon. And gravity at event horizon is infinite.

Nope. Escape velocity at event horizon is the speed of light. But the "surface gravity" so to speak, of a black hole, continues to increase until you reach the singularity.

With a sufficiently big black hole, the gravity will actually be very low- from the perspective of an object stationary relative to the singularity.

Time only slows down near the event horizon from the perspective of a distant viewer- who will see the ship appear to slow down and stop, at the horizon.

From the perspective of someone in the ship- time will pass as normal- they can enter.

http://en.wikipedia.org/wiki/Event_horizon

shawnhcorey
2013-06-21, 12:52 PM
Nope. Escape velocity at event horizon is the speed of light. But the "surface gravity" so to speak, of a black hole, continues to increase until you reach the singularity.


No, you cannot reach the singularity once the black hole is formed. As you approach the event horizon, time slows down. Because of this, GR says that the acceleration at the event horizon is infinite.

hamishspence
2013-06-21, 12:56 PM
Check the article I linked to:

Likewise, any object approaching the horizon from the observer's side appears to slow down and never quite pass through the horizon, with its image becoming more and more redshifted as time elapses. The traveling object, however, experiences no strange effects and does, in fact, pass through the horizon in a finite amount of proper time.

EDIT: The "Black Hole" article says the same:
http://en.wikipedia.org/wiki/Black_hole

To a distant observer, clocks near a black hole appear to tick more slowly than those further away from the black hole. Due to this effect, known as gravitational time dilation, an object falling into a black hole appears to slow down as it approaches the event horizon, taking an infinite time to reach it. At the same time, all processes on this object slow down, for a fixed outside observer, causing emitted light to appear redder and dimmer, an effect known as gravitational redshift. Eventually, at a point just before it reaches the event horizon, the falling object becomes so dim that it can no longer be seen.

On the other hand, an observer falling into a black hole does not notice any of these effects as he crosses the event horizon. According to his own clock, he crosses the event horizon after a finite time without noting any singular behaviour. In particular, he is unable to determine exactly when he crosses it, as it is impossible to determine the location of the event horizon from local observations.

And these are consistant with the university textbooks I've read.

shawnhcorey
2013-06-21, 01:04 PM
No, GR is about reality, not appearances. If GR says time for an object appears to slow down as it approaches the event horizon, then it actually slows down. Does not just appear to, does.

hamishspence
2013-06-21, 01:07 PM
I believe you are misunderstanding something about general relativity.

Could you point us to the place you read that said it was impossible for something to enter a black hole?

the_druid_droid
2013-06-21, 01:10 PM
Meh. FTL is all magic space unicorns anyway. Caring about whether it's quantum or wormhole or extra dimension based is like getting kvetchy that somebody named their magic space unicorn Mr. Sparklemane instead of Prancytoes.


(Which suggests the solution of having your rockets pulled by actual magic space unicorns.)

Admittedly, the quantum business is just a pet peeve of mine.

And besides, the best magical space unicorn name is clearly Heavenly Nostrils.


No, GR is about reality, not appearances. If GR says time for an object appears to slow down as it approaches the event horizon, then it actually slows down. Does not just appear to, does.

Relativity, both SR and GR, is all about frames of reference and the fact that times and distances in different frames will be measured differently by observers in those distinct frames. Hence the relativity part. You seem to still be proposing the idea of an absolute frame, which went out with the dishwater in the late 19th century and the Michelson-Morley experiments.

Soras Teva Gee
2013-06-21, 01:30 PM
Meh. FTL is all magic space unicorns anyway. Caring about whether it's quantum or wormhole or extra dimension based is like getting kvetchy that somebody named their magic space unicorn Mr. Sparklemane instead of Prancytoes.


(Which suggests the solution of having your rockets pulled by actual magic space unicorns.)

I'm extra kvetchy I guess.... those are both terrible pony names.



That said you are correct in that Hawking radiation is insignificant compared to radiation generated by accelerating matter in the accretion disk and magnetic interactions. The only case in which hawking radiation is likely to be measurable is when a black hole evaporates, which do to the time scales involved would only be seen in the smallest of black holes.

Well they would technically be evaporating all the time wouldn't they. Universe keeps expanding an all that. Its just like looking for a single dim streetlight by viewing NYC from the Moon.

Presumably not all black holes have accretion disks or otherwise are actively sucking in large quantities of matter where that may change. Just good luck finding one under such conditions at all.

shawnhcorey
2013-06-21, 01:30 PM
Relativity, both SR and GR, is all about frames of reference and the fact that times and distances in different frames will be measured differently by observers in those distinct frames. Hence the relativity part. You seem to still be proposing the idea of an absolute frame, which went out with the dishwater in the late 19th century and the Michelson-Morley experiments.

No, quite the opposite. Observed frames are what real happens. If GR says time moves slower and slower for an object as it approaches the event horizon, then that's what really happens. If you never observe it passing the event horizon, then it never does. With Relativity, observation is reality.

warty goblin
2013-06-21, 01:31 PM
Admittedly, the quantum business is just a pet peeve of mine.

It does get abused to hell and back, rather like nanotechnology. Although occasionally the abuse allows for some fun things; I rather enjoyed the quantum probabilistically immortal Achilles from Ilium for instance.


And besides, the best magical space unicorn name is clearly Heavenly Nostrils.
I stand in awe. Though Starmane is also a worthy beast.

hamishspence
2013-06-21, 01:34 PM
If you never observe it passing the event horizon, then it never does.

Ask an actual physicist- and I believe they will tell you that you are mistaken.

These bits from the above article:

Observers falling into a Schwarzschild black hole (i.e., non-rotating and not charged) cannot avoid being carried into the singularity, once they cross the event horizon.

When they reach the singularity, they are crushed to infinite density and their mass is added to the total of the black hole. Before that happens, they will have been torn apart by the growing tidal forces in a process sometimes referred to as spaghettification or the "noodle effect"

appear to contradict you. And while Wikipedia has its flaws- it's usually pretty accurate about this sort of thing.

shawnhcorey
2013-06-21, 01:40 PM
Ask an actual physicist- and I believe they will tell you that you are mistaken.

And at one time, scientists thought the Sun went around the Earth.

hamishspence
2013-06-21, 01:42 PM
The point I'm making- is that the people who study general relativity- may be able to clarify for you the nature of your misconception about what you think it says.

warty goblin
2013-06-21, 01:50 PM
And at one time, scientists thought the Sun went around the Earth.

So what? Saying the opposing view may be wrong in some non-specific manner without presenting any evidence is not an argument for the correctness of your position. Particularly when those same vague possible errors could just as easily be attributed to your position.

After all, people once thought whales a type of fish. Obviously conventional evolution is wrong, and humans are more closely related to goldfish than the great apes. I mean zits are sort of like scales, right?

hamishspence
2013-06-21, 01:55 PM
If you're falling into a black hole, and looking out of it with a very powerful telescope (Let's say it's a big black hole, like the one at the centre of our galaxy) would apparent time on the "outside" appear to speed up, until everything disappears?

Soras Teva Gee
2013-06-21, 02:04 PM
And at one time, scientists thought the Sun went around the Earth.

Oh so science is wrong and you are right is it now?

Excellent. Now put up.

Now show us your math, studies, and observations that have led to your conclusion.

Not merely invoke previously deprecated theories. That we should take what "we know" with a grain of salt is wise. I'll not be surprised if/when Dark Matter and Dark Energy are worked out some ideas will look as silly as Aether theory does today. However using that we have been wrong before as evidence you are right is nonsense.

shawnhcorey
2013-06-21, 02:10 PM
The point I'm making- is that the people who study general relativity- may be able to clarify for you the nature of your misconception about what you think it says.

If Relativity was only an observation, then the Twins Paradox wouldn't work. And physicists are notorious bad at math.


If you're falling into a black hole, and looking out of it with a very powerful telescope (Let's say it's a big black hole, like the one at the centre of our galaxy) would apparent time on the "outside" appear to speed up, until everything disappears?

Yes, the expansion of the universe would speed up, not apparently speed up. Eventually, everything will be far away and Hawking radiation would vaporize the black hole. You would end up in the far, distance future with nothing around you and nothing in sight.

hamishspence
2013-06-21, 02:20 PM
Going by the descriptions of how it works on various science sites- the person would not (from their own perspective) live for ever- in fact, it would only be a few minutes or hours (depending on the size of the hole) before they hit the vicinity of the singularity, and are torn apart.

What tears them apart, is the fact that, though they are "falling" the strength of the gravity they are experiencing, varies- the end of the person nearest the singularity, will be experiencing a vastly stronger force than the far end of them.

shawnhcorey
2013-06-21, 02:21 PM
Oh so science is wrong and you are right is it now?

Excellent. Now put up.

Now show us your math, studies, and observations that have led to your conclusion.

Not merely invoke previously deprecated theories. That we should take what "we know" with a grain of salt is wise. I'll not be surprised if/when Dark Matter and Dark Energy are worked out some ideas will look as silly as Aether theory does today. However using that we have been wrong before as evidence you are right is nonsense.

From Wikipedia Event Horizon (https://en.wikipedia.org/wiki/Event_horizon#Interacting_with_an_event_horizon):

A misconception concerning event horizons, especially black hole event horizons, is that they represent an immutable surface that destroys objects that approach them. In practice, all event horizons appear to be some distance away from any observer, and objects sent towards an event horizon never appear to cross it from the sending observer's point of view (as the horizon-crossing event's light cone never intersects the observer's world line). Attempting to make an object near the horizon remain stationary with respect to an observer requires applying a force whose magnitude increases unbounded (becoming infinite) the closer it gets.

Note the part that says "...a force whose magnitude increases unbounded (becoming infinite)..." Apparently, I'm not the only one who thinks gravity goes to infinite as an object gets near the event horizon.

hamishspence
2013-06-21, 02:28 PM
No- not gravity- the force they're talking about is the force that would be needed to reach the speed of light- which is infinite.

You'd need to exert that force, in order to remain stationary relative to the singularity, at the event horizon.

Gravity itself does not become "infinite" until one reaches the singularity itself.

On the same Wikipedia page:

Once a particle is inside the horizon, moving into the hole is as inevitable as moving forward in time, and can actually be thought of as equivalent to doing so, depending on the spacetime coordinate system used.

shawnhcorey
2013-06-21, 02:48 PM
No- not gravity- the force they're talking about is the force that would be needed to reach the speed of light- which is infinite.

No, it says object, that is, something with mass. To keep a mass stationary, the force needed increases to infinite as it positioned closer to the event horizon.

And GR is not just about gravity. GR is about acceleration. It does not matter the source of the acceleration.

hamishspence
2013-06-21, 02:51 PM
Hence- it's impossible to keep the object stationary. Hence, it moves in the direction of the singularity, until it reaches the singularity (having been destroyed in the process through spaghettification- the varying strength of the gravitational force between the far end of the object and the near end).

For a supermassive black hole- this does not happen until the object is well inside the event horizon.

http://en.wikipedia.org/wiki/Supermassive_black_hole

Mando Knight
2013-06-21, 02:55 PM
I'll not be surprised if/when Dark Matter and Dark Energy are worked out some ideas will look as silly as Aether theory does today.

Including the possibility that some strange field equation pops up and both are as real as phlogiston. (There are some attempts at modifying Newtonian and Einsteinian mechanics equations to eliminate the need for such strange things)

...Actually, y'know what? I think Aether and Quintessence are cooler names than Dark Matter/Energy.

hamishspence
2013-06-21, 03:07 PM
One unusual form of FTL in Harry Harrison's Bill the Galactic Hero- is the Bloater drive.

Object is inflated to immense size.

Object is shrunk- but with one side (the side at the destination) "held"

Result- by just growing and shrinking, one has travelled.

shawnhcorey
2013-06-21, 03:41 PM
Hence- it's impossible to keep the object stationary. Hence, it moves in the direction of the singularity, until it reaches the singularity...

That article is not about the singularity, it's about the event horizon. The force increase to infinity as the object is placed nearer and nearer the event horizon. That's the second time you misinterpreted the article. If you don't believe me, sit down and do the math yourself. The equations for GR are straight forward.

Anima
2013-06-21, 03:42 PM
Actually time dilatation at an event horizon is relatively easy to explain. The closer an event occurs to the horizon the longer the light from that event takes to reach the distant observer. (Why this happens is the complicated part.)
But since the speed of light is constant and the distance is approximately constant as well time has to change. (Relatively is pretty much noticing all the strange that result from the constance of c.)

Now when the event approaches the horizon the time its light takes to reach the observer approaches infinity. Since the horizon is by definition the point where light can't escape you will never observe the event crossing the horizon even if you wait until the end of the universe.

The odd part is that this dilatation only occurs between the reference frames of the observer and the event. For the event time continues just normal. Though if the event would observe the observer, the observer would seem to be accelerated.

That's also what no special frame of reference means, both observations are correct in their respective frame of reference.
(It's not relativity without a headache.)

For SF writers who are interested in scientific accuracy I can recommend this site http://www.projectrho.com/public_html/rocket/index.php.
It also has a bit on FTL, though only at gunpoint. That said I agree which the position that any FTL device should suit the story instead of reality. Thinking what universe you'd like to have and the design the FTL device you need is probably the better way around. That is, if you don't want to explore the ramifications of current FTL proposals of course.

Disclaimer, this explanation is wrong, but relatively useful. Any puns are pure coincidence.

shawnhcorey
2013-06-21, 03:46 PM
The odd part is that this dilatation only occurs between the reference frames of the observer and the event. For the event time continues just normal. Though if the event would observe the observer, the observer would seem to be accelerated.

Yes, as you approach the event horizon, time for the rest of the universe will speed up. If you were to turn around and move away from the event horizon, you would find yourself in the distance future.

the_druid_droid
2013-06-21, 05:39 PM
I stand in awe. Though Starmane is also a worthy beast.

Well, I did have a little help (http://www.gocomics.com/heavenly-nostrils)


If Relativity was only an observation, then the Twins Paradox wouldn't work. And physicists are notorious bad at math.

AHAHAHAHAHAHAHAHAHAHAHAAAAAAAAAAAAAAhahahahahahaha haaaaaaaaa... ha ha ha haaaaa...

There are so many things wrong with that statement I don't even know where to start.

shawnhcorey
2013-06-21, 05:43 PM
There are so many things wrong with that statement I don't even know where to start.

There were two statements there. Are you a physicist?

The Extinguisher
2013-06-21, 05:50 PM
Time doesn't dilate for the object travelling towards the event horizon. In fact, we'd actually say his frame is the "proper time" compared to the distant observer.

Which in the end, it doesn't matter. Because for the object he passes through the event horizon, and for the observer he doesn't ever quite get there. And both of them are true.
It's not a difficult concept to grasp (other than the fact that it is very difficult to get, I guess)

shawnhcorey
2013-06-21, 06:08 PM
Time doesn't dilate for the object travelling towards the event horizon. In fact, we'd actually say his frame is the "proper time" compared to the distant observer.

There is no preferred frame of reference. There is no proper observer.


Which in the end, it doesn't matter. Because for the object he passes through the event horizon, and for the observer he doesn't ever quite get there. And both of them are true.
It's not a difficult concept to grasp (other than the fact that it is very difficult to get, I guess)

Nothing passes through the event horizon. For any object approaching the event horizon, at some point, time slows down faster than it accelerates. It never reaches the event horizon.

Anima
2013-06-21, 06:37 PM
Nothing passes through the event horizon. For any object approaching the event horizon, at some point, time slows down faster than it accelerates. It never reaches the event horizon.
For the things that approach the event horizon time doesn't slow down though. The time dilatation occurs between the events and the observers reference frame. From the events perspective it's the observer that behaves strange.

Or in other words, while nothing ever crosses the event horizon in the observers frame, it does in the events frame.

shawnhcorey
2013-06-21, 06:39 PM
Or in other words, while nothing ever crosses the event horizon in the observers frame, it does in the events frame.

No, time speeds up in the rest of the universe and eventually the black hole vaporizes due to Hawking radiation.

Anima
2013-06-21, 06:48 PM
No, time speeds up in the rest of the universe and eventually the black hole vaporizes due to Hawking radiation.

Not quite, only distant objects (those farther outside of the gravity well) are speed up. So while you can enjoy a nice compressed view of time on the outside before the rules of physics break down. Time doesn't change for the black hole itself, so it won't evaporate before you had the chance to cross the horizon.

The Extinguisher
2013-06-21, 08:19 PM
There is no preferred frame of reference. There is no proper observer.


Yes I know that. But, when comparing reference frames, you need to have a Δt and a Δt' between two events, and we give the name "proper" time to the Δt that is observed by the clock that passes through both events. It's just convention.

Maryring
2013-06-21, 10:10 PM
It seems to me that before we get any further into generation ships and such--to say nothing of terraformed moons--it would be helpful if Maryring could tell us a little more about what she wants her particular starship to do.
Warfare. Exploration. Colonization. I want to create a world where Space Pirates roam and drink Space grog from acid resistant Space pewter cups. Sure, it may be a bit nitpicky to worry about scientific accuracy in a world where mining corporations attack research stations in order to make hefty profits, but I do want a story that is as solid as possible, both in research and storytelling.

Ravens_cry
2013-06-22, 12:16 AM
Warfare. Exploration. Colonization. I want to create a world where Space Pirates roam and drink Space grog from acid resistant Space pewter cups. Sure, it may be a bit nitpicky to worry about scientific accuracy in a world where mining corporations attack research stations in order to make hefty profits, but I do want a story that is as solid as possible, both in research and storytelling.
How does that help them make hefty profits? Now, space piracy could be an option if you your FTL uses a 'gate' system, creating choke points and there was not a strong government military space presence to barricade them. Perhaps they are some ancient found artifact no one really understands, let alone owns.
For in-system piracy, large cargo craft would be moving on the local equivalent of the ITN (http://en.wikipedia.org/wiki/Interplanetary_Transport_Network) as that's cheapest. This creates predictability, potentially giving pirates a place to swoop in.

the_druid_droid
2013-06-22, 12:30 AM
There is no preferred frame of reference. There is no proper observer.

Proper time (http://en.wikipedia.org/wiki/Proper_time) is a technical term in relativity. It has nothing to do with preferred frames, and simply refers to the time measured in the frame of the moving object under consideration.

In that sense "proper" has the archaic meaning of "belonging to oneself" and not the current connotation of "correct" or "appropriate".

hamishspence
2013-06-22, 01:57 AM
That article is not about the singularity, it's about the event horizon. The force increase to infinity as the object is placed nearer and nearer the event horizon. That's the second time you misinterpreted the article. If you don't believe me, sit down and do the math yourself.

Try the basic formula for gravity for a 1 million solar mass point source object. The force will increase, from zero (infinitely far away from the object) to infinity- at the point source, not the event horizon.

Otherwise, speaking of a huge differential between gravitational force at the head, and gravitational force at the feet, would not make sense.

http://en.wikipedia.org/wiki/Supermassive_black_hole


the tidal forces in the vicinity of the event horizon are significantly weaker. Since the central singularity is so far away from the horizon, a hypothetical astronaut traveling towards the black hole center would not experience significant tidal force until very deep into the black hole.

I did astrophysics at university- I do know what I'm talking about.

There's another detailed essay on black holes here:
http://essayweb.net/astronomy/blackhole_3.shtml

Important bits:

Consider two observers, Alice and Bob. Alice is approaching the black hole, while Bob sits back far away, at a safe distance from it. As Alice gets closer to the black hole, it will appear to Bob that she is slowing down. The closer she gets to the black hole, the more she will seem to slow down to Bob, until she reaches the event horizon, at which point she will appear to stop. Bob will never see her enter the event horizon - it will look to him like she's stuck at the event horizon forever.

This is a thought experiment, of course, since in reality if Alice were to approach that close, she would be torn apart into atoms and there would be nothing for Bob to observe anyway. But let's disregard that for the moment. Now, from Alice's perspective, time does not slow down. She does in fact reach the event horizon, pass through, eventually to merge with the gravitational singularity inside. For Alice, the whole trip would probably take no more than a few seconds.

The radiation from a black hole is called Hawking radiation, and is in a perfect black body spectrum. The amount of radiation is proportional to surface gravity of the black hole (surface gravity being the gravity at the event horizon). Since smaller black holes have much higher surface gravity than large ones, they are expected to produce a lot more radiation than larger ones.

So, it's not a case of them both having "infinite surface gravity at the event horizon" - the bigger black hole's surface gravity is actually a lot lower, despite it being impossible to escape it.

shawnhcorey
2013-06-22, 07:39 AM
Try the basic formula for gravity for a 1 million solar mass point source object. The force will increase, from zero (infinitely far away from the object) to infinity- at the point source, not the event horizon.

Then gravity cannot get out of the black hole. The event horizon is the point where the escape velocity is the speed of light. Since gravity travels at the speed of light, it cannot escape. Black holes would have no gravity.

noparlpf
2013-06-22, 09:43 AM
Quick question. How do black holes emit Hawking radiation? Is it something that appears somewhere outside the event horizon, or does it originate inside, and somehow break the speed of light to escape?


Then gravity cannot get out of the black hole. The event horizon is the point where the escape velocity is the speed of light. Since gravity travels at the speed of light, it cannot escape. Black holes would have no gravity.

What? That doesn't make any sense.

shawnhcorey
2013-06-22, 09:53 AM
Quick question. How do black holes emit Hawking radiation? Is it something that appears somewhere outside the event horizon, or does it originate inside, and somehow break the speed of light to escape?

An anomaly happens just outside the event horizon where a quantum and an anti-quantum are created. One falls into the black hole, the other flies away. When it hits another quantum, it emits radiation that is characteristic of Hawking radiation. That's how we can detect it, by its signature radiation.



Then gravity cannot get out of the black hole. The event horizon is the point where the escape velocity is the speed of light. Since gravity travels at the speed of light, it cannot escape. Black holes would have no gravity.
What? That doesn't make any sense.

Precisely. Gravity simply can not come from inside a black hole. Or it travels faster than light. Choose one.

Emmerask
2013-06-22, 10:00 AM
Quick question. How do black holes emit Hawking radiation? Is it something that appears somewhere outside the event horizon, or does it originate inside, and somehow break the speed of light to escape?


One simplified model would be happening just outside the event horizon:

A particle and antiparticle is "created" near the event horizon
five possibilities:
1)they annihilate each other instantly
2)both "fall" in the blackhole (no change in mass)
3)both escape the gravitational pull (no change in mass)
4)the antiparticle falls in annihilating a particle, while the particle escapes = a net loss in mass for the blackhole.
5)the particle falls in while the antiparticle escaping (= netgain in mass) ?
Somehow this is never discussed but wouldn´t that happen as often as 4) ie blackholes dont evaporate at all?

The other model happens just inside the event horizon:

like the first model but the particle tunnels (infinite speed) outside the event horizon, in that case it somewhat "breaks" the speed of light limit

At least thats how I as a none physicist understand hawking radiation I might be very wrong there :smallsmile:

Anima
2013-06-22, 10:00 AM
Wikipedia tries to give an explanation: https://en.wikipedia.org/wiki/Hawking_radiation


Warfare. Exploration. Colonization. I want to create a world where Space Pirates roam and drink Space grog from acid resistant Space pewter cups. Sure, it may be a bit nitpicky to worry about scientific accuracy in a world where mining corporations attack research stations in order to make hefty profits, but I do want a story that is as solid as possible, both in research and storytelling.
For space pirates to exist we have to rule out all FTL methods that rely on mega structures. Even jump points are problematic since all choke points make undetected travel impossible.
For warfare we need to be able to intercept FTL capable ships, either during FTL or by detecting them in FTL and then intercept them in normal space.

A drive that takes you to a different dimension where travel times are reduced would be a possible solution. Added points if it behaves a bit like underwater. Fortified system would have listening posts that can detect ships in this hyperspace, but it would be difficult to identify single ships.
It'd also allow the pirates to sneak up to their prey in hyperspace, think about the potential for drama. Add a device that can take ships out of hyperspace and a cooldown/warm up period for the drive and you can have a normal exiting space battle as well.

It's only a poorly thought out suggestion though. The important point is to first ask what drive you need for your setting and then think about the consequences of that drive.

Mando Knight
2013-06-22, 10:07 AM
Precisely. Gravity simply can not come from inside a black hole. Or it travels faster than light. Choose one.

I believe the prevailing theory is that gravity doesn't travel, it just is.

noparlpf
2013-06-22, 10:09 AM
An anomaly happens just outside the event horizon where a quantum and an anti-quantum are created. One falls into the black hole, the other flies away. When it hits another quantum, it emits radiation that is characteristic of Hawking radiation. That's how we can detect it, by its signature radiation.

Oh, that stuff. I remember reading something about quantum entanglement and black holes being on fire, and they mentioned that.


Precisely. Gravity simply can not come from inside a black hole. Or it travels faster than light. Choose one.

Uh, I'm pretty sure forces don't travel to the things they affect. They're just there.

shawnhcorey
2013-06-22, 10:14 AM
I believe the prevailing theory is that gravity doesn't travel, it just is.


Uh, I'm pretty sure forces don't travel to the things they affect. They're just there.

Then how would a gravity field change? At the speed of light, at least, that's what Einstein thought.

Emmerask
2013-06-22, 10:20 AM
Uh, I'm pretty sure forces don't travel to the things they affect. They're just there.

Afaik gravity is the only fundamental force without (MAYBE) an exchange particle.

shawnhcorey
2013-06-22, 10:25 AM
Afaik gravity is the only fundamental force without (MAYBE) an exchange particle.

That's what String Theory is all about, giving gravity a quantum.

Ronnoc
2013-06-22, 10:52 AM
One simplified model would be happening just outside the event horizon:

A particle and antiparticle is "created" near the event horizon
five possibilities:
1)they annihilate each other instantly
2)both "fall" in the blackhole (no change in mass)
3)both escape the gravitational pull (no change in mass)
4)the antiparticle falls in annihilating a particle, while the particle escapes = a net loss in mass for the blackhole.
5)the particle falls in while the antiparticle escaping (= netgain in mass) ?
Somehow this is never discussed but wouldn´t that happen as often as 4) ie blackholes dont evaporate at all?

The other model happens just inside the event horizon:

like the first model but the particle tunnels (infinite speed) outside the event horizon, in that case it somewhat "breaks" the speed of light limit

At least thats how I as a none physicist understand hawking radiation I might be very wrong there :smallsmile:

Your resolution between steps four and five are handled by conservation of energy from the perspective of an outside observer the process that you just described is the black hole emitting particles. These particles have mass and therefore they have energy. This energy has to come from somewhere in this case the rest mass of the black hole.



Well they would technically be evaporating all the time wouldn't they. Universe keeps expanding an all that. Its just like looking for a single dim streetlight by viewing NYC from the Moon.

Presumably not all black holes have accretion disks or otherwise are actively sucking in large quantities of matter where that may change. Just good luck finding one under such conditions at all.

Yes, you are right we should have hawking radiation being admitted all the time. What I meant when I said evaporation was the endpoint of black hole evaporation which should result in a large, characteristic burst of gamma rays. That said this evaporation is still dim compared to stellar radiation and would have to occur locally to be measurable.


If Relativity was only an observation, then the Twins Paradox wouldn't work. And physicists are notorious bad at math.


...The twins paradox doesn't work. It uses a non-inertial reference frame invalidating it from the standpoint of special relativity.


Then gravity cannot get out of the black hole. The event horizon is the point where the escape velocity is the speed of light. Since gravity travels at the speed of light, it cannot escape. Black holes would have no gravity.

From the perspective of general relativity gravity does not depend on force carriers to translate, it is a purely geometric result of matter. Now Gravity waves do indeed propagate at the speed of light but the field does not. For an interesting metaphor to the problem you should look up the difference between group and phase velocities. While quantum mechanics in general and string theory in particular does in fact support the existence of a force carrier for gravitation at this time no workable theory of quantum gravity has been devised.

Mando Knight
2013-06-22, 11:03 AM
Also, the escape velocity is for objects leaving the influence of a gravity well. If gravity needs to travel faster than the escape velocity to propagate itself, then what the hell is pulling gravity back?

(Also, the theoretical graviton is a massless particle)

shawnhcorey
2013-06-22, 11:17 AM
...The twins paradox doesn't work. It uses a non-inertial reference frame invalidating it from the standpoint of special relativity.

The Twin Paradox has been proven by experiment (http://en.wikipedia.org/wiki/Hafele%E2%80%93Keating_experiment).


Now Gravity waves do indeed propagate at the speed of light but the field does not.

Mathematically, the field and changes to it propagate at the same rate.


Also, the escape velocity is for objects leaving the influence of a gravity well. If gravity needs to travel faster than the escape velocity to propagate itself, then what the hell is pulling gravity back?

That's correct. Half the stuff you read about black holes contradicts the other half or other accepted theories.

The Extinguisher
2013-06-22, 11:37 AM
The twin paradox isn't really a paradox (other than the sense that I guess having two twins be different ages is a paradox?), and it should really be called the Twin Weird Thing.

Because it isn't logically inconsistent. It's just weird.

Mando Knight
2013-06-22, 11:39 AM
You're saying that gravity needs to be able to escape its own influence to exert itself.

The Extinguisher
2013-06-22, 11:44 AM
Yeah, that's a pretty solid point. Either gravity doesn't have a particle associated with it, in which case it's just a warping of space-time and everything is fine, or it does, and the particle doesn't interact with itself and everything is still fine.

noparlpf
2013-06-22, 12:01 PM
Either way, black holes obviously exert gravity, so gravity is clearly not bounded by the speed of light.

shawnhcorey
2013-06-22, 12:37 PM
You're saying that gravity needs to be able to escape its own influence to exert itself.


Yeah, that's a pretty solid point. Either gravity doesn't have a particle associated with it, in which case it's just a warping of space-time and everything is fine, or it does, and the particle doesn't interact with itself and everything is still fine.


Either way, black holes obviously exert gravity, so gravity is clearly not bounded by the speed of light.

I'm saying the gravity of a black hole looks like this (http://www.wolframalpha.com/input/?i=y%3D-1%2F%28x^2-1.57%29), where 1.57 represents the radius of the event horizon. (I had to give it a real value for the graph to come out right.)

Note that inside the black hole, gravity is a repulsive force (its sign is positive). Everything inside is pushed outward toward the event horizon. There is an infinite force inside the event horizon pushing outward against an infinite force pushing inward.

Mando Knight
2013-06-22, 12:41 PM
I'm saying the gravity of a black hole looks like this (http://www.wolframalpha.com/input/?i=y%3D-1%2F%28x^2-1.57%29), where 1.57 represents the radius of the event horizon. (I had to give it a real value for the graph to come out right.)

Note that inside the black hole, gravity is a repulsive force (its sign is positive). Everything inside is pushed outward toward the event horizon. There is an infinite force inside the event horizon pushing outward against an infinite force pushing inward.

That's ridiculous. Do you have any source for this?

shawnhcorey
2013-06-22, 12:51 PM
That's ridiculous. Do you have any source for this?

Yes: Attempting to make an object near the horizon remain stationary with respect to an observer requires applying a force whose magnitude increases unbounded (becoming infinite) the closer it gets. (https://en.wikipedia.org/wiki/Event_horizon#Interacting_with_an_event_horizon)

BTW, if you insult me once more, I'll inform the moderators.

The Extinguisher
2013-06-22, 01:03 PM
Yes, that's the force with respect to the observer.

For the event, he can pass through just okay, without ever applying another force, because from his reference frame, the event horizon isn't anything significant. He just keeps accelerating towards the singularity, which is where all the mass is.

If the mass was at the event horizon, as you seem to suggest, it would be a lot different.

Anima
2013-06-22, 01:42 PM
Yes: Attempting to make an object near the horizon remain stationary with respect to an observer requires applying a force whose magnitude increases unbounded (becoming infinite) the closer it gets. (https://en.wikipedia.org/wiki/Event_horizon#Interacting_with_an_event_horizon)

BTW, if you insult me once more, I'll inform the moderators.
Of course it does, since the escape velocity approaches c, so the energy to accelerate the object to escape velocity approaches infinity. This doesn't require infinite gravity.

And no one insulted you so far, even if it's tempting considering your tone.

shawnhcorey
2013-06-22, 01:47 PM
And no one insulted you so far, even if it's tempting considering your tone.

Calling a person's ideas names is calling the person names.

shawnhcorey
2013-06-22, 01:49 PM
Of course it does, since the escape velocity approaches c, so the energy to accelerate the object to escape velocity approaches infinity. This doesn't require infinite gravity.

The quote states that the force, not the energy or speed, goes to infinite. That is, gravity goes to infinite.

razark
2013-06-22, 01:57 PM
Calling a person's ideas names is calling the person names.
No it's not.

Even a smart person can hold dumb ideas.

Anima
2013-06-22, 02:00 PM
If somebody said that only an idiot could hold a position, that would be an insult. Saying that an idea is bad or ridiculous, while not exactly a nice way to put it, is not an insult.


The quote states that the force, not the energy or speed, goes to infinite. That is, gravity goes to infinite.
Well maybe because all three are related.
Since energy is force times time and force is mass times acceleration.
So when force goes to infinity, energy does as well. (That's easy to see. No matter how short the time, you'll never reach infinite force with finite energy.)
And speed never goes to infinite, it can only go to c.

The reverse is also true, in finite time infinite energy always results in infinite force.

shawnhcorey
2013-06-22, 02:02 PM
No it's not.

Even a smart person can hold dumb ideas.

Yes, I agree.

noparlpf
2013-06-22, 02:28 PM
Of course it does, since the escape velocity approaches c, so the energy to accelerate the object to escape velocity approaches infinity. This doesn't require infinite gravity.

Isn't that considered to be mass dilation?

Anima
2013-06-22, 02:44 PM
Isn't that considered to be mass dilation?
Yes, though it's a bit more complex. Since the rest mass (that's what we consider in Newtonian mechanics) is an invariant.
But trough E = mc˛ we get a second meaning for mass, the relativistic mass which is a measure of an objects energy.
So an object with infinite energy would have infinite gravity, which is where the current theories break down.

My point was that this doesn't require infinite gravity, only enough to raise the escape velocity to c. Though my own understanding is insufficient to really understand what's going on.

Soras Teva Gee
2013-06-22, 02:51 PM
My point was that this doesn't require infinite gravity, only enough to raise the escape velocity to c. Though my own understanding is insufficient to really understand what's going on.

I think people might be having a bit of trouble because in other contexts lightspeed as a sort of absolute limit is hit on hard enough its starts to sound like there shouldn't be anything that exceeds lightspeed in any context.

Anima
2013-06-22, 03:12 PM
I think people might be having a bit of trouble because in other contexts lightspeed as a sort of absolute limit is hit on hard enough its starts to sound like there shouldn't be anything that exceeds lightspeed in any context.

I'm not sure where anything exceeds the limit of light speed so far. At the event horizon the escape velocity exceeds c, since nothing can be faster nothing can escape the black hole. So no matter where you are going your destination will be the black hole. Basically at that point any action you could take will have the result of having intimate contact with a singularity.

While changes in a gravitational field might propagate at c, that has nothing to do with the gravitational influence of the black hole. Though I have to admit that I can't really explain it, my knowledge is limited to the basics.
An useful picture might be that space is the floor you're walking on. Gravity tilts the floor. If the tilt is step enough you can't reach the other end of the room any more. But that doesn't affect whatever is tilting the floor.
Hope that helps a bit.

Soras Teva Gee
2013-06-22, 03:20 PM
I'm not sure where anything exceeds the limit of light speed so far. At the event horizon the escape velocity exceeds c,

Highlighting for effect.

I know thats not right I'm talking about at the sort of lazy thought process I myself use. Since I'm not a physicist and just beat at the edges of this I feel a certain disconnect there. I know its nothing more then an attempt at oversimplification of universe but its there.

Anima
2013-06-22, 03:32 PM
Highlighting for effect.

I know thats not right I'm talking about at the sort of lazy thought process I myself use. Since I'm not a physicist and just beat at the edges of this I feel a certain disconnect there. I know its nothing more then an attempt at oversimplification of universe but its there.

Ah, thanks. I seriously didn't see the problem.
While we can not exceed c we can of course talk about situations that requite to exceed c. For example a camping trip to Sirius and back in your next summer vacation. This would obviously require FTL. Of course it's impossible as far as we know.

When I (and others) talk about escape velocities exceeding c, it's the same as with the camping trip. It's not possible, but we can still think about it.
Maybe it would be better to say to say that no escape velocity exists at and behind the event horizon. (Which sounds odd to me this time.) Would that help?

Though truth be told I find that problem a bit hilarious considering the thread title. :smallwink:

The Extinguisher
2013-06-22, 05:10 PM
At work on my phone, so ill be brief. Talking about forces here is wrong. To accelerate a massive particle to escape a black hole at the event horizon is impossible. The amount of energy needed increases without bounds. It makes far more sense to talk about it in terms of energy because the forces involved are irelavent. The force due to gravity from the black hole is finite, but the potential energy well is impossible to escape, evem if there was an outward force that was larger than the black holes gravity

You slow down, but you'll still end up at the singularity in the end.

shawnhcorey
2013-06-22, 05:25 PM
You slow down, but you'll still end up at the singularity in the end.

No, you won't. There are only two solutions:

1. Gravity is create by the singularity and travels faster than light (or it can't escape the event horizon).

2. Gravity is created at the event horizon and travels at the speed of light.

If you choose #1, black holes have no gravity.

Anima
2013-06-22, 05:44 PM
Since a singularity is the point of infinite mass and gravity and the event horizon is not an object, this should be pretty obvious by now.
Here is the wikipedia article to Speed of Gravity (https://en.wikipedia.org/wiki/Speed_of_gravity)

@ The Extinguisher
Agreed, but the wikipedia article used force. And an infinite increase of energy is the same as infinite energy.

shawnhcorey
2013-06-22, 05:56 PM
Since a singularity is the point of infinite mass and gravity and the event horizon is not an object, this should be pretty obvious by now.
Here is the wikipedia article to Speed of Gravity (https://en.wikipedia.org/wiki/Speed_of_gravity)

@ The Extinguisher
Agreed, but the wikipedia article used force. And an infinite increase of energy is the same as infinite energy.

The singularity has finite mass, the mass of the star that created it.

There are no objects in physics, or rather, objects are part of Newton's physics. Both Relativity and Quantum Mechanics is about fields and energy.

For example, both an electron and a photon are point sources. If so, they can never collide. But both are electromagnetic phenomenon. When we say the collide we mean that their fields interact with each other and causes changes to each, such as a change in velocity. Objects are not solid things but fields and energy.

The Extinguisher
2013-06-22, 06:04 PM
General relatively is incompatible with quantum electrodynamics, so apply one to the other is wrong thinking.

Assuming gravity is carried by particles, those travel at c being massless
But a virtual graviton wouldnt need to interact with itself, so gravity works fine


Again, though, QED does not work with GR so this discussion is pointless.

Anima
2013-06-22, 06:10 PM
The singularity has finite mass, the mass of the star that created it.

You are correct, the singularity is the point of infinite density. And I need more sleep.


There are no objects in physics, or rather, objects are part of Newton's physics. Both Relativity and Quantum Mechanics is about fields and energy.

For example, both an electron and a photon are point sources. If so, they can never collide. But both are electromagnetic phenomenon. When we say the collide we mean that their fields interact with each other and causes changes to each, such as a change in velocity. Objects are not solid things but fields and energy.

That's a question of definition and what phenomena you are trying to describe. It doesn't matter in this case because the event horizon is still only a theoretical construct, it's the distance where all possible paths end at the singularity.

shawnhcorey
2013-06-22, 06:16 PM
That's a question of definition and what phenomena you are trying to describe. It doesn't matter in this case because the event horizon is still only a theoretical construct, it's the distance where all possible paths end at the singularity.

Sorry, there are many, many, many theoretical constructs. Just use your imagination. But only a few of them are real.

So, you agree. Gravity cannot escape from the event horizon.

Anima
2013-06-22, 06:22 PM
You know, I'm actually studying information science, I'm familiar with abstract objects.

Gravity does not need to escape the event horizon in the first place. Without it the horizon wouldn't exist, or to go even further the gravitational field is the horizon. (Well the horizon is an attribute of the field.)

shawnhcorey
2013-06-22, 06:25 PM
You know, I'm actually studying information science, I'm familiar with abstract objects.

Gravity does not need to escape the event horizon in the first place. Without it the horizon wouldn't exist, or to go even further the gravitational field is the horizon. (Well the horizon is an attribute of the field.)

That's right, it's a paradox. If GR causes the event horizon, then it must be wrong because gravity must travel faster than light or black holes have no gravity.

Of course, if you assume the event horizon creates the gravity, the paradox goes away.

Anima
2013-06-22, 06:33 PM
That's right, it's a paradox. If GR causes the event horizon, then it must be wrong because gravity must travel faster than light or black holes have no gravity.

Of course, if you assume the event horizon creates the gravity, the paradox goes away.

I have the nagging suspicion that while this discussion is not in a deadlock it's not alive either.
You just responded to my statement that the speed of gravity is completely unimportant with agreement. And then you assert that his shows that the speed of gravity causes a paradox.

Is there any source for your statement that the event horizon creates gravity? I'd like to see it.

The Extinguisher
2013-06-22, 06:40 PM
The problem with that is that a shell does not behave as a point mass. And the event horizon only makes sense if the black hole is a point mass.

Im not sure why you think that a gravity needs to be affected by gravity though. I would like yo see some evidence to that effect though.

shawnhcorey
2013-06-22, 06:52 PM
Im not sure why you think that a gravity needs to be affected by gravity though. I would like yo see some evidence to that effect though.

GR changes space. If light can't go fast enough to escape, nothing else can because space is warped by gravity. Gravity effects gravity because it changes the shape of space. See General Relativity (http://en.wikipedia.org/wiki/General_relativity).

The Extinguisher
2013-06-22, 07:08 PM
Thats combinging QED with GR though. Which are incompatible. Gravity warping spacetime is GR, which means it doesnt need to worry about interacting with itself because its just a property of mass. If theres a particle that can be slowed down, thats QED which means it doesnt warp spacetime so it doesnt slow down.

Unless you managed to unify GR and QED, you're coming at this with conflicting assumptions.

shawnhcorey
2013-06-22, 07:32 PM
Thats combinging QED with GR though. Which are incompatible. Gravity warping spacetime is GR, which means it doesnt need to worry about interacting with itself because its just a property of mass. If theres a particle that can be slowed down, thats QED which means it doesnt warp spacetime so it doesnt slow down.

Unless you managed to unify GR and QED, you're coming at this with conflicting assumptions.

Sorry, quanta that have mass do warp space. All mass does regardless of how small it is.

Mando Knight
2013-06-22, 07:37 PM
Sorry, quanta that have mass do warp space. All mass does regardless of how small it is.

Then if gravity is caused by gravitons, then the graviton itself doesn't warp space because it would be a massless spin-2 boson.

Maryring
2013-06-22, 08:20 PM
How does that help them make hefty profits? Now, space piracy could be an option if you your FTL uses a 'gate' system, creating choke points and there was not a strong government military space presence to barricade them. Perhaps they are some ancient found artifact no one really understands, let alone owns.
For in-system piracy, large cargo craft would be moving on the local equivalent of the ITN (http://en.wikipedia.org/wiki/Interplanetary_Transport_Network) as that's cheapest. This creates predictability, potentially giving pirates a place to swoop in.
Oh it's a long story. Remind me to tell it to you some day... when it is done.


Wikipedia tries to give an explanation: https://en.wikipedia.org/wiki/Hawking_radiation


For space pirates to exist we have to rule out all FTL methods that rely on mega structures. Even jump points are problematic since all choke points make undetected travel impossible.
For warfare we need to be able to intercept FTL capable ships, either during FTL or by detecting them in FTL and then intercept them in normal space.

A drive that takes you to a different dimension where travel times are reduced would be a possible solution. Added points if it behaves a bit like underwater. Fortified system would have listening posts that can detect ships in this hyperspace, but it would be difficult to identify single ships.
It'd also allow the pirates to sneak up to their prey in hyperspace, think about the potential for drama. Add a device that can take ships out of hyperspace and a cooldown/warm up period for the drive and you can have a normal exiting space battle as well.

It's only a poorly thought out suggestion though. The important point is to first ask what drive you need for your setting and then think about the consequences of that drive.
You just inspired me a great, great deal. Thank you. Thank all of you. This thread's a goldmine of inspiration for me.

warty goblin
2013-06-22, 08:45 PM
For piracy, I'd suggest a form of FTL closer to sixteenth and seventeenth century (or earlier) sailing vessels. That is while a ship is in transit, it is essentially out of contact with the mainland, and can only be detected over a relatively small radius on the ocean. In space terms, once you go FTL you can't talk to things not going FTL, and your FTL signature is only detectable over a (relatively) small area. This means that, once away from planets, pirates can go about pirating in privacy. Make planets rare enough, and travel times long enough, along with some gubbins about hyperspace currents or tachyon winds or whatever and you've got shipping routes, which is to say ideal places for a bit of high-stars robbery.



Ships need ports, and baring some weird stuff, spaceships are no different. Somewhere to sell off booty - which is absolutely vital, else why be a pirate? - refit the ship, refuel, and stock up on space-grog. Since most centralized governments don't like other actors infringing on their monopoly of taking people's stuff under threat of bodily harm, you're going to need lawless or neutral territory of some nature for your space pirates to perform these vital functions far away from the arm of the law in order to avoid swinging from a space-gibbet.

Refitting spaceships is probably something you need a reasonable level of industry to accomplish, so a just founded colonial backwater isn't your best bet. I'd go with a region of space rendered lawless and devoid of a strong hegemonic power following the collapse of some multi-planetary government. This nets you reasonably developed planetary economies, the shipyards necessary for maintaining pirate vessels, the gobs of military hardware lying around needed for the actual pirating bits, and an economy happy to look the other way when Cap'n Eyepatch shows up with a big load of space-rum ready to sell. Add in something - alien invasions, social unrest, whatever - to keep the governments of other regions of space too busy to step in and fill the power vacuum, and your pirates now have a relatively safe base of operations.

Let the redistribution of wealth via space-cutlass commence.

Anima
2013-06-22, 09:05 PM
Hyperspace is also an interesting location for pirate bases. If you can send a ship into it, why not an entire base?
The pirate captains would buy rendezvous points from the base, without them it would be basically impossible to find. And the space patrol would hunt for the coordinates.

A hostile power also creates good opportunities to turn your bounty into money. It's also a good source for military hardware, as long as you stay on the right side of the border.

factotum
2013-06-23, 01:16 AM
For piracy, I'd suggest a form of FTL closer to sixteenth and seventeenth century (or earlier) sailing vessels. That is while a ship is in transit, it is essentially out of contact with the mainland, and can only be detected over a relatively small radius on the ocean. In space terms, once you go FTL you can't talk to things not going FTL, and your FTL signature is only detectable over a (relatively) small area.

Or maybe even have it that they can send an *object* at FTL velocities, but not a signal--that would then lead to all sorts of consequences. For a start, you could never detect a ship travelling FTL until it transitioned to normal space, and such a ship would be in a communications blackout. In addition, you'd have ships all over the place acting as fast messengers, because it would be faster in almost all cases to send a physical ship to the destination with the message than it would be to transmit it.

That's what I think of when I think about "hard" FTL--you define the rules of the FTL, then figure out how everything else works based on that.

Ravens_cry
2013-06-23, 11:38 AM
That's what I think of when I think about "hard" FTL--you define the rules of the FTL, then figure out how everything else works based on that.
Exactly!:smallbiggrin:

Flickerdart
2013-06-23, 08:31 PM
In addition, you'd have ships all over the place acting as fast messengers, because it would be faster in almost all cases to send a physical ship to the destination with the message than it would be to transmit it.
These would probably be automated probes, though. In any case, I would imagine that getting energy to tunnel through space would be easier than getting matter to tunnel through space.

noparlpf
2013-06-23, 08:34 PM
These would probably be automated probes, though. In any case, I would imagine that getting energy to tunnel through space would be easier than getting matter to tunnel through space.

Considering energy signals can already be transmitted at about light speed.

Ravens_cry
2013-06-23, 10:36 PM
What if your ship goes to a different 'space' when it goes FTL and has to reopen the door on the other side? In that case, couriers, which could simply be automated craft filled with some form of computer memory, would be the only way to do FTL communications.

factotum
2013-06-24, 02:11 AM
Considering energy signals can already be transmitted at about light speed.

But we're in the realm of fantasy when talking about FTL, so you can pretty much make things up as you go along. Besides, Ravens_cry just came up with a perfectly good justification for FTL travel working in the way I described... :smallwink:

(Seriously: radio signals and what-have-you don't propagate at "about" light speed, they propagate *at* light speed because they're electromagnetic waves. The only reason they go slower is due to travelling through our atmosphere--out in space, they go the full C. Transmitting data FTL would require a different mechanism).

the_druid_droid
2013-06-24, 02:16 AM
One last thing on the black hole debate that's been going on - I believe I've found an answer from a highly reliable source (unless anyone wants to impugn the Cosmology department at UC Berkeley).

Here (http://cosmology.berkeley.edu/Education/BHfaq.html#q3) is the relevant link, and I'll quote a bit too (emphasis mine):


For a very large black hole like the one [at the center of the Milky Way], the tidal forces are not really noticeable until you get within about 600,000 kilometers of the center. Note that this is after you've crossed the horizon. If you were falling into a smaller black hole, say one that weighed as much as the Sun, tidal forces would start to make you quite uncomfortable when you were about 6000 kilometers away from the center, and you would have been torn apart by them long before you crossed the horizon. (That's why we decided to let you jump into a big black hole instead of a small one: we wanted you to survive at least until you got inside.)